Find the best upper bound of $varepsilon$ such that $g(x) = 1 + varepsilon f(x) > 0$, $f(x)$ bounded on a...












0












$begingroup$


Consider a smooth function $f : [0, 1] to mathbb{R}$. Moreover, consider:



$$g(x) = 1 + varepsilon f(x),$$



for $x in [0, 1]$ and some $varepsilon > 0$.



Which are the conditions on $varepsilon$ which guarantee that $g(x) > 0$? I guess the answer is something in the form:



$$0 < varepsilon < M,$$



for some $M > 0$.





My attempt



Let $[0, 1] = I^- cup I^+$, where:





  • $f(x) < 0 ~forall x in I^-$;


  • $f(x) geq 0 ~forall x in I^+$.


For $x in I^+$, then $g(x) > 0$.



For $x in I^-$, since function $f$ is (smooth, then) bounded, then:



$$exists F > 0 : f(x)> -F ~forall x in I^-.$$



Therefore:



$$g(x) = 1 + varepsilon f(x) > 1 - varepsilon F ~forall x in I^-.$$



If $varepsilon < frac{1}{F}$, then:



$$1 - varepsilon F > 0,$$



and hence



$$g(x) > 0 ~forall x in [0, 1].$$





Questions



Is there some easies way to achieve this kind of results?



Is there any better upper bound for $varepsilon$?










share|cite|improve this question









$endgroup$

















    0












    $begingroup$


    Consider a smooth function $f : [0, 1] to mathbb{R}$. Moreover, consider:



    $$g(x) = 1 + varepsilon f(x),$$



    for $x in [0, 1]$ and some $varepsilon > 0$.



    Which are the conditions on $varepsilon$ which guarantee that $g(x) > 0$? I guess the answer is something in the form:



    $$0 < varepsilon < M,$$



    for some $M > 0$.





    My attempt



    Let $[0, 1] = I^- cup I^+$, where:





    • $f(x) < 0 ~forall x in I^-$;


    • $f(x) geq 0 ~forall x in I^+$.


    For $x in I^+$, then $g(x) > 0$.



    For $x in I^-$, since function $f$ is (smooth, then) bounded, then:



    $$exists F > 0 : f(x)> -F ~forall x in I^-.$$



    Therefore:



    $$g(x) = 1 + varepsilon f(x) > 1 - varepsilon F ~forall x in I^-.$$



    If $varepsilon < frac{1}{F}$, then:



    $$1 - varepsilon F > 0,$$



    and hence



    $$g(x) > 0 ~forall x in [0, 1].$$





    Questions



    Is there some easies way to achieve this kind of results?



    Is there any better upper bound for $varepsilon$?










    share|cite|improve this question









    $endgroup$















      0












      0








      0





      $begingroup$


      Consider a smooth function $f : [0, 1] to mathbb{R}$. Moreover, consider:



      $$g(x) = 1 + varepsilon f(x),$$



      for $x in [0, 1]$ and some $varepsilon > 0$.



      Which are the conditions on $varepsilon$ which guarantee that $g(x) > 0$? I guess the answer is something in the form:



      $$0 < varepsilon < M,$$



      for some $M > 0$.





      My attempt



      Let $[0, 1] = I^- cup I^+$, where:





      • $f(x) < 0 ~forall x in I^-$;


      • $f(x) geq 0 ~forall x in I^+$.


      For $x in I^+$, then $g(x) > 0$.



      For $x in I^-$, since function $f$ is (smooth, then) bounded, then:



      $$exists F > 0 : f(x)> -F ~forall x in I^-.$$



      Therefore:



      $$g(x) = 1 + varepsilon f(x) > 1 - varepsilon F ~forall x in I^-.$$



      If $varepsilon < frac{1}{F}$, then:



      $$1 - varepsilon F > 0,$$



      and hence



      $$g(x) > 0 ~forall x in [0, 1].$$





      Questions



      Is there some easies way to achieve this kind of results?



      Is there any better upper bound for $varepsilon$?










      share|cite|improve this question









      $endgroup$




      Consider a smooth function $f : [0, 1] to mathbb{R}$. Moreover, consider:



      $$g(x) = 1 + varepsilon f(x),$$



      for $x in [0, 1]$ and some $varepsilon > 0$.



      Which are the conditions on $varepsilon$ which guarantee that $g(x) > 0$? I guess the answer is something in the form:



      $$0 < varepsilon < M,$$



      for some $M > 0$.





      My attempt



      Let $[0, 1] = I^- cup I^+$, where:





      • $f(x) < 0 ~forall x in I^-$;


      • $f(x) geq 0 ~forall x in I^+$.


      For $x in I^+$, then $g(x) > 0$.



      For $x in I^-$, since function $f$ is (smooth, then) bounded, then:



      $$exists F > 0 : f(x)> -F ~forall x in I^-.$$



      Therefore:



      $$g(x) = 1 + varepsilon f(x) > 1 - varepsilon F ~forall x in I^-.$$



      If $varepsilon < frac{1}{F}$, then:



      $$1 - varepsilon F > 0,$$



      and hence



      $$g(x) > 0 ~forall x in [0, 1].$$





      Questions



      Is there some easies way to achieve this kind of results?



      Is there any better upper bound for $varepsilon$?







      algebra-precalculus proof-verification upper-lower-bounds






      share|cite|improve this question













      share|cite|improve this question











      share|cite|improve this question




      share|cite|improve this question










      asked Dec 2 '18 at 16:01









      the_candymanthe_candyman

      8,91632145




      8,91632145






















          2 Answers
          2






          active

          oldest

          votes


















          2












          $begingroup$

          It's easier than you think. You just need that your function is continuous. Indeed, since $f$ is continuous, for the extreme value theorem $f$ must attain a maximum $M$ and a minimum $m$.

          Then (SPOILER)




          $$
          g(x)geq 0 quad forall xin [0,1] Leftrightarrow
          1+varepsilon f(x)geq 0 Leftrightarrow 1+varepsilon mgeq 0
          $$

          If $m=0$, it's obvious that $g(x)>0$, otherwise we select $varepsilon >0$ such that
          $varepsilongeq frac{-1}{m}$







          share|cite|improve this answer









          $endgroup$





















            1












            $begingroup$

            If $f(x) ge 0$, then $1+epsilon f(x) > 0$ for all values of $epsilon$.



            If $f(x) < 0$, then



            begin{align}
            g(x) > 0
            &iff 1 + epsilon f(x) > 0 \
            &iff epsilon f(x) > -1 \
            &iff epsilon < -dfrac{1}{f(x)}
            end{align}



            So $displaystyle epsilon = -max_{f(x)<0} dfrac{1}{f(x)}$






            share|cite|improve this answer











            $endgroup$













            • $begingroup$
              Thanks a lot for the answer. Then, can I take the lowest between $$-min_{f(x)<0} dfrac{1}{f(x)}$$ and $$-frac{1}{min_{f(x)<0} f(x)},$$ right?
              $endgroup$
              – the_candyman
              Dec 2 '18 at 19:30












            • $begingroup$
              I had to rethink my answer and change it. That should be $-frac{1}{max_{f(x)<0} f(x)},$
              $endgroup$
              – steven gregory
              Dec 2 '18 at 19:40










            • $begingroup$
              In that case, your answer is the same of the one provided by Andrea Cavaliere.
              $endgroup$
              – the_candyman
              Dec 2 '18 at 19:49










            • $begingroup$
              Yes. I just did the logic differenly.
              $endgroup$
              – steven gregory
              Dec 2 '18 at 20:06











            Your Answer





            StackExchange.ifUsing("editor", function () {
            return StackExchange.using("mathjaxEditing", function () {
            StackExchange.MarkdownEditor.creationCallbacks.add(function (editor, postfix) {
            StackExchange.mathjaxEditing.prepareWmdForMathJax(editor, postfix, [["$", "$"], ["\\(","\\)"]]);
            });
            });
            }, "mathjax-editing");

            StackExchange.ready(function() {
            var channelOptions = {
            tags: "".split(" "),
            id: "69"
            };
            initTagRenderer("".split(" "), "".split(" "), channelOptions);

            StackExchange.using("externalEditor", function() {
            // Have to fire editor after snippets, if snippets enabled
            if (StackExchange.settings.snippets.snippetsEnabled) {
            StackExchange.using("snippets", function() {
            createEditor();
            });
            }
            else {
            createEditor();
            }
            });

            function createEditor() {
            StackExchange.prepareEditor({
            heartbeatType: 'answer',
            autoActivateHeartbeat: false,
            convertImagesToLinks: true,
            noModals: true,
            showLowRepImageUploadWarning: true,
            reputationToPostImages: 10,
            bindNavPrevention: true,
            postfix: "",
            imageUploader: {
            brandingHtml: "Powered by u003ca class="icon-imgur-white" href="https://imgur.com/"u003eu003c/au003e",
            contentPolicyHtml: "User contributions licensed under u003ca href="https://creativecommons.org/licenses/by-sa/3.0/"u003ecc by-sa 3.0 with attribution requiredu003c/au003e u003ca href="https://stackoverflow.com/legal/content-policy"u003e(content policy)u003c/au003e",
            allowUrls: true
            },
            noCode: true, onDemand: true,
            discardSelector: ".discard-answer"
            ,immediatelyShowMarkdownHelp:true
            });


            }
            });














            draft saved

            draft discarded


















            StackExchange.ready(
            function () {
            StackExchange.openid.initPostLogin('.new-post-login', 'https%3a%2f%2fmath.stackexchange.com%2fquestions%2f3022805%2ffind-the-best-upper-bound-of-varepsilon-such-that-gx-1-varepsilon-fx%23new-answer', 'question_page');
            }
            );

            Post as a guest















            Required, but never shown

























            2 Answers
            2






            active

            oldest

            votes








            2 Answers
            2






            active

            oldest

            votes









            active

            oldest

            votes






            active

            oldest

            votes









            2












            $begingroup$

            It's easier than you think. You just need that your function is continuous. Indeed, since $f$ is continuous, for the extreme value theorem $f$ must attain a maximum $M$ and a minimum $m$.

            Then (SPOILER)




            $$
            g(x)geq 0 quad forall xin [0,1] Leftrightarrow
            1+varepsilon f(x)geq 0 Leftrightarrow 1+varepsilon mgeq 0
            $$

            If $m=0$, it's obvious that $g(x)>0$, otherwise we select $varepsilon >0$ such that
            $varepsilongeq frac{-1}{m}$







            share|cite|improve this answer









            $endgroup$


















              2












              $begingroup$

              It's easier than you think. You just need that your function is continuous. Indeed, since $f$ is continuous, for the extreme value theorem $f$ must attain a maximum $M$ and a minimum $m$.

              Then (SPOILER)




              $$
              g(x)geq 0 quad forall xin [0,1] Leftrightarrow
              1+varepsilon f(x)geq 0 Leftrightarrow 1+varepsilon mgeq 0
              $$

              If $m=0$, it's obvious that $g(x)>0$, otherwise we select $varepsilon >0$ such that
              $varepsilongeq frac{-1}{m}$







              share|cite|improve this answer









              $endgroup$
















                2












                2








                2





                $begingroup$

                It's easier than you think. You just need that your function is continuous. Indeed, since $f$ is continuous, for the extreme value theorem $f$ must attain a maximum $M$ and a minimum $m$.

                Then (SPOILER)




                $$
                g(x)geq 0 quad forall xin [0,1] Leftrightarrow
                1+varepsilon f(x)geq 0 Leftrightarrow 1+varepsilon mgeq 0
                $$

                If $m=0$, it's obvious that $g(x)>0$, otherwise we select $varepsilon >0$ such that
                $varepsilongeq frac{-1}{m}$







                share|cite|improve this answer









                $endgroup$



                It's easier than you think. You just need that your function is continuous. Indeed, since $f$ is continuous, for the extreme value theorem $f$ must attain a maximum $M$ and a minimum $m$.

                Then (SPOILER)




                $$
                g(x)geq 0 quad forall xin [0,1] Leftrightarrow
                1+varepsilon f(x)geq 0 Leftrightarrow 1+varepsilon mgeq 0
                $$

                If $m=0$, it's obvious that $g(x)>0$, otherwise we select $varepsilon >0$ such that
                $varepsilongeq frac{-1}{m}$








                share|cite|improve this answer












                share|cite|improve this answer



                share|cite|improve this answer










                answered Dec 2 '18 at 16:34









                Andrea CavaliereAndrea Cavaliere

                1156




                1156























                    1












                    $begingroup$

                    If $f(x) ge 0$, then $1+epsilon f(x) > 0$ for all values of $epsilon$.



                    If $f(x) < 0$, then



                    begin{align}
                    g(x) > 0
                    &iff 1 + epsilon f(x) > 0 \
                    &iff epsilon f(x) > -1 \
                    &iff epsilon < -dfrac{1}{f(x)}
                    end{align}



                    So $displaystyle epsilon = -max_{f(x)<0} dfrac{1}{f(x)}$






                    share|cite|improve this answer











                    $endgroup$













                    • $begingroup$
                      Thanks a lot for the answer. Then, can I take the lowest between $$-min_{f(x)<0} dfrac{1}{f(x)}$$ and $$-frac{1}{min_{f(x)<0} f(x)},$$ right?
                      $endgroup$
                      – the_candyman
                      Dec 2 '18 at 19:30












                    • $begingroup$
                      I had to rethink my answer and change it. That should be $-frac{1}{max_{f(x)<0} f(x)},$
                      $endgroup$
                      – steven gregory
                      Dec 2 '18 at 19:40










                    • $begingroup$
                      In that case, your answer is the same of the one provided by Andrea Cavaliere.
                      $endgroup$
                      – the_candyman
                      Dec 2 '18 at 19:49










                    • $begingroup$
                      Yes. I just did the logic differenly.
                      $endgroup$
                      – steven gregory
                      Dec 2 '18 at 20:06
















                    1












                    $begingroup$

                    If $f(x) ge 0$, then $1+epsilon f(x) > 0$ for all values of $epsilon$.



                    If $f(x) < 0$, then



                    begin{align}
                    g(x) > 0
                    &iff 1 + epsilon f(x) > 0 \
                    &iff epsilon f(x) > -1 \
                    &iff epsilon < -dfrac{1}{f(x)}
                    end{align}



                    So $displaystyle epsilon = -max_{f(x)<0} dfrac{1}{f(x)}$






                    share|cite|improve this answer











                    $endgroup$













                    • $begingroup$
                      Thanks a lot for the answer. Then, can I take the lowest between $$-min_{f(x)<0} dfrac{1}{f(x)}$$ and $$-frac{1}{min_{f(x)<0} f(x)},$$ right?
                      $endgroup$
                      – the_candyman
                      Dec 2 '18 at 19:30












                    • $begingroup$
                      I had to rethink my answer and change it. That should be $-frac{1}{max_{f(x)<0} f(x)},$
                      $endgroup$
                      – steven gregory
                      Dec 2 '18 at 19:40










                    • $begingroup$
                      In that case, your answer is the same of the one provided by Andrea Cavaliere.
                      $endgroup$
                      – the_candyman
                      Dec 2 '18 at 19:49










                    • $begingroup$
                      Yes. I just did the logic differenly.
                      $endgroup$
                      – steven gregory
                      Dec 2 '18 at 20:06














                    1












                    1








                    1





                    $begingroup$

                    If $f(x) ge 0$, then $1+epsilon f(x) > 0$ for all values of $epsilon$.



                    If $f(x) < 0$, then



                    begin{align}
                    g(x) > 0
                    &iff 1 + epsilon f(x) > 0 \
                    &iff epsilon f(x) > -1 \
                    &iff epsilon < -dfrac{1}{f(x)}
                    end{align}



                    So $displaystyle epsilon = -max_{f(x)<0} dfrac{1}{f(x)}$






                    share|cite|improve this answer











                    $endgroup$



                    If $f(x) ge 0$, then $1+epsilon f(x) > 0$ for all values of $epsilon$.



                    If $f(x) < 0$, then



                    begin{align}
                    g(x) > 0
                    &iff 1 + epsilon f(x) > 0 \
                    &iff epsilon f(x) > -1 \
                    &iff epsilon < -dfrac{1}{f(x)}
                    end{align}



                    So $displaystyle epsilon = -max_{f(x)<0} dfrac{1}{f(x)}$







                    share|cite|improve this answer














                    share|cite|improve this answer



                    share|cite|improve this answer








                    edited Dec 2 '18 at 19:39

























                    answered Dec 2 '18 at 19:27









                    steven gregorysteven gregory

                    18.2k32258




                    18.2k32258












                    • $begingroup$
                      Thanks a lot for the answer. Then, can I take the lowest between $$-min_{f(x)<0} dfrac{1}{f(x)}$$ and $$-frac{1}{min_{f(x)<0} f(x)},$$ right?
                      $endgroup$
                      – the_candyman
                      Dec 2 '18 at 19:30












                    • $begingroup$
                      I had to rethink my answer and change it. That should be $-frac{1}{max_{f(x)<0} f(x)},$
                      $endgroup$
                      – steven gregory
                      Dec 2 '18 at 19:40










                    • $begingroup$
                      In that case, your answer is the same of the one provided by Andrea Cavaliere.
                      $endgroup$
                      – the_candyman
                      Dec 2 '18 at 19:49










                    • $begingroup$
                      Yes. I just did the logic differenly.
                      $endgroup$
                      – steven gregory
                      Dec 2 '18 at 20:06


















                    • $begingroup$
                      Thanks a lot for the answer. Then, can I take the lowest between $$-min_{f(x)<0} dfrac{1}{f(x)}$$ and $$-frac{1}{min_{f(x)<0} f(x)},$$ right?
                      $endgroup$
                      – the_candyman
                      Dec 2 '18 at 19:30












                    • $begingroup$
                      I had to rethink my answer and change it. That should be $-frac{1}{max_{f(x)<0} f(x)},$
                      $endgroup$
                      – steven gregory
                      Dec 2 '18 at 19:40










                    • $begingroup$
                      In that case, your answer is the same of the one provided by Andrea Cavaliere.
                      $endgroup$
                      – the_candyman
                      Dec 2 '18 at 19:49










                    • $begingroup$
                      Yes. I just did the logic differenly.
                      $endgroup$
                      – steven gregory
                      Dec 2 '18 at 20:06
















                    $begingroup$
                    Thanks a lot for the answer. Then, can I take the lowest between $$-min_{f(x)<0} dfrac{1}{f(x)}$$ and $$-frac{1}{min_{f(x)<0} f(x)},$$ right?
                    $endgroup$
                    – the_candyman
                    Dec 2 '18 at 19:30






                    $begingroup$
                    Thanks a lot for the answer. Then, can I take the lowest between $$-min_{f(x)<0} dfrac{1}{f(x)}$$ and $$-frac{1}{min_{f(x)<0} f(x)},$$ right?
                    $endgroup$
                    – the_candyman
                    Dec 2 '18 at 19:30














                    $begingroup$
                    I had to rethink my answer and change it. That should be $-frac{1}{max_{f(x)<0} f(x)},$
                    $endgroup$
                    – steven gregory
                    Dec 2 '18 at 19:40




                    $begingroup$
                    I had to rethink my answer and change it. That should be $-frac{1}{max_{f(x)<0} f(x)},$
                    $endgroup$
                    – steven gregory
                    Dec 2 '18 at 19:40












                    $begingroup$
                    In that case, your answer is the same of the one provided by Andrea Cavaliere.
                    $endgroup$
                    – the_candyman
                    Dec 2 '18 at 19:49




                    $begingroup$
                    In that case, your answer is the same of the one provided by Andrea Cavaliere.
                    $endgroup$
                    – the_candyman
                    Dec 2 '18 at 19:49












                    $begingroup$
                    Yes. I just did the logic differenly.
                    $endgroup$
                    – steven gregory
                    Dec 2 '18 at 20:06




                    $begingroup$
                    Yes. I just did the logic differenly.
                    $endgroup$
                    – steven gregory
                    Dec 2 '18 at 20:06


















                    draft saved

                    draft discarded




















































                    Thanks for contributing an answer to Mathematics Stack Exchange!


                    • Please be sure to answer the question. Provide details and share your research!

                    But avoid



                    • Asking for help, clarification, or responding to other answers.

                    • Making statements based on opinion; back them up with references or personal experience.


                    Use MathJax to format equations. MathJax reference.


                    To learn more, see our tips on writing great answers.




                    draft saved


                    draft discarded














                    StackExchange.ready(
                    function () {
                    StackExchange.openid.initPostLogin('.new-post-login', 'https%3a%2f%2fmath.stackexchange.com%2fquestions%2f3022805%2ffind-the-best-upper-bound-of-varepsilon-such-that-gx-1-varepsilon-fx%23new-answer', 'question_page');
                    }
                    );

                    Post as a guest















                    Required, but never shown





















































                    Required, but never shown














                    Required, but never shown












                    Required, but never shown







                    Required, but never shown

































                    Required, but never shown














                    Required, but never shown












                    Required, but never shown







                    Required, but never shown







                    Popular posts from this blog

                    Biblatex bibliography style without URLs when DOI exists (in Overleaf with Zotero bibliography)

                    ComboBox Display Member on multiple fields

                    Is it possible to collect Nectar points via Trainline?